Partition of Unity in Spivak's Calculus on Manifolds












13












$begingroup$


I have a question about partitions of unity specifically in the book Calculus on Manifolds by Spivak. In case 1 for the proof of existence of partition of unity, why is there a need for the function $f$? The set $Phi = {varphi_1, dotsc, varphi_n}$ looks like is already the desired partition of unity. Following is the theorem and proof. Only Case 1 in the proof is relevant.



enter image description hereenter image description here










share|cite|improve this question











$endgroup$












  • $begingroup$
    Should I include pictures of the two pages with the theorem and proof?
    $endgroup$
    – Pratyush Sarkar
    Aug 24 '13 at 13:24










  • $begingroup$
    That would certainly help; or at least type in the relevant details (what are the $phi_i, f$ and what is the proposed partition of unity).
    $endgroup$
    – Anthony Carapetis
    Aug 24 '13 at 14:00










  • $begingroup$
    I am reading calculus on manifolds too. And as I came across this proof today, I have exactly the same doubt. Glad that I find this post.
    $endgroup$
    – A. Chu
    Jul 10 '15 at 14:16










  • $begingroup$
    On what set is $sigma$ define din the second case? Because I do not see how how defining $phi / sigma$ gives a partition of unity, or even how the division makes sense.
    $endgroup$
    – vaoy
    Apr 18 '18 at 10:51










  • $begingroup$
    @vaoy $Phi_i$ is a partition of unity subordinate to $mathcal O_i$, so we can assume $varphi in Phi_i$ is defined on $A$ and zero outside $operatorname{int}(A_{i + 1}) - A_{i - 2}$. And $sigma$ is also defined on $A$ and as explained, the sum in the definition is at most finite for all $x in A$ and necessarily positive, so division is possible. Note that $sigma$ is just a scaling factor to make sure the new $varphi'$ sum up to $1$ at each $x in A$. This scaling is required because although $Phi_i$ is a partition of unity, $Phi_j$ for $j = i + 1, i - 1$ can interfere with $Phi_i$.
    $endgroup$
    – Pratyush Sarkar
    Apr 24 '18 at 18:00


















13












$begingroup$


I have a question about partitions of unity specifically in the book Calculus on Manifolds by Spivak. In case 1 for the proof of existence of partition of unity, why is there a need for the function $f$? The set $Phi = {varphi_1, dotsc, varphi_n}$ looks like is already the desired partition of unity. Following is the theorem and proof. Only Case 1 in the proof is relevant.



enter image description hereenter image description here










share|cite|improve this question











$endgroup$












  • $begingroup$
    Should I include pictures of the two pages with the theorem and proof?
    $endgroup$
    – Pratyush Sarkar
    Aug 24 '13 at 13:24










  • $begingroup$
    That would certainly help; or at least type in the relevant details (what are the $phi_i, f$ and what is the proposed partition of unity).
    $endgroup$
    – Anthony Carapetis
    Aug 24 '13 at 14:00










  • $begingroup$
    I am reading calculus on manifolds too. And as I came across this proof today, I have exactly the same doubt. Glad that I find this post.
    $endgroup$
    – A. Chu
    Jul 10 '15 at 14:16










  • $begingroup$
    On what set is $sigma$ define din the second case? Because I do not see how how defining $phi / sigma$ gives a partition of unity, or even how the division makes sense.
    $endgroup$
    – vaoy
    Apr 18 '18 at 10:51










  • $begingroup$
    @vaoy $Phi_i$ is a partition of unity subordinate to $mathcal O_i$, so we can assume $varphi in Phi_i$ is defined on $A$ and zero outside $operatorname{int}(A_{i + 1}) - A_{i - 2}$. And $sigma$ is also defined on $A$ and as explained, the sum in the definition is at most finite for all $x in A$ and necessarily positive, so division is possible. Note that $sigma$ is just a scaling factor to make sure the new $varphi'$ sum up to $1$ at each $x in A$. This scaling is required because although $Phi_i$ is a partition of unity, $Phi_j$ for $j = i + 1, i - 1$ can interfere with $Phi_i$.
    $endgroup$
    – Pratyush Sarkar
    Apr 24 '18 at 18:00
















13












13








13


2



$begingroup$


I have a question about partitions of unity specifically in the book Calculus on Manifolds by Spivak. In case 1 for the proof of existence of partition of unity, why is there a need for the function $f$? The set $Phi = {varphi_1, dotsc, varphi_n}$ looks like is already the desired partition of unity. Following is the theorem and proof. Only Case 1 in the proof is relevant.



enter image description hereenter image description here










share|cite|improve this question











$endgroup$




I have a question about partitions of unity specifically in the book Calculus on Manifolds by Spivak. In case 1 for the proof of existence of partition of unity, why is there a need for the function $f$? The set $Phi = {varphi_1, dotsc, varphi_n}$ looks like is already the desired partition of unity. Following is the theorem and proof. Only Case 1 in the proof is relevant.



enter image description hereenter image description here







real-analysis multivariable-calculus






share|cite|improve this question















share|cite|improve this question













share|cite|improve this question




share|cite|improve this question








edited Apr 24 '18 at 10:46









Arkya Chatterjee

541315




541315










asked Aug 24 '13 at 13:23









Pratyush SarkarPratyush Sarkar

2,7801127




2,7801127












  • $begingroup$
    Should I include pictures of the two pages with the theorem and proof?
    $endgroup$
    – Pratyush Sarkar
    Aug 24 '13 at 13:24










  • $begingroup$
    That would certainly help; or at least type in the relevant details (what are the $phi_i, f$ and what is the proposed partition of unity).
    $endgroup$
    – Anthony Carapetis
    Aug 24 '13 at 14:00










  • $begingroup$
    I am reading calculus on manifolds too. And as I came across this proof today, I have exactly the same doubt. Glad that I find this post.
    $endgroup$
    – A. Chu
    Jul 10 '15 at 14:16










  • $begingroup$
    On what set is $sigma$ define din the second case? Because I do not see how how defining $phi / sigma$ gives a partition of unity, or even how the division makes sense.
    $endgroup$
    – vaoy
    Apr 18 '18 at 10:51










  • $begingroup$
    @vaoy $Phi_i$ is a partition of unity subordinate to $mathcal O_i$, so we can assume $varphi in Phi_i$ is defined on $A$ and zero outside $operatorname{int}(A_{i + 1}) - A_{i - 2}$. And $sigma$ is also defined on $A$ and as explained, the sum in the definition is at most finite for all $x in A$ and necessarily positive, so division is possible. Note that $sigma$ is just a scaling factor to make sure the new $varphi'$ sum up to $1$ at each $x in A$. This scaling is required because although $Phi_i$ is a partition of unity, $Phi_j$ for $j = i + 1, i - 1$ can interfere with $Phi_i$.
    $endgroup$
    – Pratyush Sarkar
    Apr 24 '18 at 18:00




















  • $begingroup$
    Should I include pictures of the two pages with the theorem and proof?
    $endgroup$
    – Pratyush Sarkar
    Aug 24 '13 at 13:24










  • $begingroup$
    That would certainly help; or at least type in the relevant details (what are the $phi_i, f$ and what is the proposed partition of unity).
    $endgroup$
    – Anthony Carapetis
    Aug 24 '13 at 14:00










  • $begingroup$
    I am reading calculus on manifolds too. And as I came across this proof today, I have exactly the same doubt. Glad that I find this post.
    $endgroup$
    – A. Chu
    Jul 10 '15 at 14:16










  • $begingroup$
    On what set is $sigma$ define din the second case? Because I do not see how how defining $phi / sigma$ gives a partition of unity, or even how the division makes sense.
    $endgroup$
    – vaoy
    Apr 18 '18 at 10:51










  • $begingroup$
    @vaoy $Phi_i$ is a partition of unity subordinate to $mathcal O_i$, so we can assume $varphi in Phi_i$ is defined on $A$ and zero outside $operatorname{int}(A_{i + 1}) - A_{i - 2}$. And $sigma$ is also defined on $A$ and as explained, the sum in the definition is at most finite for all $x in A$ and necessarily positive, so division is possible. Note that $sigma$ is just a scaling factor to make sure the new $varphi'$ sum up to $1$ at each $x in A$. This scaling is required because although $Phi_i$ is a partition of unity, $Phi_j$ for $j = i + 1, i - 1$ can interfere with $Phi_i$.
    $endgroup$
    – Pratyush Sarkar
    Apr 24 '18 at 18:00


















$begingroup$
Should I include pictures of the two pages with the theorem and proof?
$endgroup$
– Pratyush Sarkar
Aug 24 '13 at 13:24




$begingroup$
Should I include pictures of the two pages with the theorem and proof?
$endgroup$
– Pratyush Sarkar
Aug 24 '13 at 13:24












$begingroup$
That would certainly help; or at least type in the relevant details (what are the $phi_i, f$ and what is the proposed partition of unity).
$endgroup$
– Anthony Carapetis
Aug 24 '13 at 14:00




$begingroup$
That would certainly help; or at least type in the relevant details (what are the $phi_i, f$ and what is the proposed partition of unity).
$endgroup$
– Anthony Carapetis
Aug 24 '13 at 14:00












$begingroup$
I am reading calculus on manifolds too. And as I came across this proof today, I have exactly the same doubt. Glad that I find this post.
$endgroup$
– A. Chu
Jul 10 '15 at 14:16




$begingroup$
I am reading calculus on manifolds too. And as I came across this proof today, I have exactly the same doubt. Glad that I find this post.
$endgroup$
– A. Chu
Jul 10 '15 at 14:16












$begingroup$
On what set is $sigma$ define din the second case? Because I do not see how how defining $phi / sigma$ gives a partition of unity, or even how the division makes sense.
$endgroup$
– vaoy
Apr 18 '18 at 10:51




$begingroup$
On what set is $sigma$ define din the second case? Because I do not see how how defining $phi / sigma$ gives a partition of unity, or even how the division makes sense.
$endgroup$
– vaoy
Apr 18 '18 at 10:51












$begingroup$
@vaoy $Phi_i$ is a partition of unity subordinate to $mathcal O_i$, so we can assume $varphi in Phi_i$ is defined on $A$ and zero outside $operatorname{int}(A_{i + 1}) - A_{i - 2}$. And $sigma$ is also defined on $A$ and as explained, the sum in the definition is at most finite for all $x in A$ and necessarily positive, so division is possible. Note that $sigma$ is just a scaling factor to make sure the new $varphi'$ sum up to $1$ at each $x in A$. This scaling is required because although $Phi_i$ is a partition of unity, $Phi_j$ for $j = i + 1, i - 1$ can interfere with $Phi_i$.
$endgroup$
– Pratyush Sarkar
Apr 24 '18 at 18:00






$begingroup$
@vaoy $Phi_i$ is a partition of unity subordinate to $mathcal O_i$, so we can assume $varphi in Phi_i$ is defined on $A$ and zero outside $operatorname{int}(A_{i + 1}) - A_{i - 2}$. And $sigma$ is also defined on $A$ and as explained, the sum in the definition is at most finite for all $x in A$ and necessarily positive, so division is possible. Note that $sigma$ is just a scaling factor to make sure the new $varphi'$ sum up to $1$ at each $x in A$. This scaling is required because although $Phi_i$ is a partition of unity, $Phi_j$ for $j = i + 1, i - 1$ can interfere with $Phi_i$.
$endgroup$
– Pratyush Sarkar
Apr 24 '18 at 18:00












3 Answers
3






active

oldest

votes


















6












$begingroup$

I was looking back at my question today for some reason and immediately saw why the function $f$ is required. Although $psi_i$ is smooth with compact support in $U_i$, the functions $varphi_i$ can only be defined on $U$ where $sum_{i = 1}^npsi_i > 0$. The problem is that $varphi_i$ usually does not go to zero at the boundary of $operatorname{supp}(psi_i)$ (much less smoothly extend to zero outside the boundary). You can see this near the boundary of a $operatorname{supp}(psi_i)$ which is away from all other $operatorname{supp}(psi_j)$. Near this boundary $varphi_i(x) = frac{psi_i(x)}{psi_i(x)} = 1$. The solution is to use a cutoff function $f$ which forces everything to smoothly go to $0$ near the boundary of $U$.






share|cite|improve this answer











$endgroup$













  • $begingroup$
    I think you mean $varphi_{i}$ (not $psi_{i}$) does not usually go to zero at the boundary of $text{supp}(psi_{i})$.
    $endgroup$
    – fourierwho
    May 31 '16 at 16:37










  • $begingroup$
    @fourierwho Yes. Thanks for the correction.
    $endgroup$
    – Pratyush Sarkar
    Jul 25 '16 at 23:14










  • $begingroup$
    @PratyushSarkar, the fact of $phi_i$ doesn't smoothly extend to zero outside the boundary of $operatorname{supp} (psi_i)$ doesn't contradicts the fact of $psi_i$ are $C^{infty}$ by the way was built? If your answer is not, so I think that I didn't understand the purpose of $f$.
    $endgroup$
    – George
    Jan 29 '17 at 18:28












  • $begingroup$
    @George No there is no contradiction. What I explained is, although $psi_i$ obviously extends to $0$ smoothly outside $operatorname{supp} (psi_i)$, the biggest set where we can define $varphi_i$ is only on $U = bigcup_{i = 1}^n operatorname{supp} (psi_i)$ and it is possible that near the boundary of $U$ which, say, coincides with $operatorname{supp} (psi_i)$ and is away from all other $operatorname{supp} (psi_j)$, the value of $varphi_i(x)$ tends to $1$.
    $endgroup$
    – Pratyush Sarkar
    Feb 17 '17 at 5:56












  • $begingroup$
    More explicitly, we can easily think of (and draw) an example where $B_i = partial U setminus left(bigcup_{j neq i} operatorname{supp} (psi_j)right)$ is nonempty and in that case we have $lim_{x to b} varphi_i(x) = 1$ for any $b in B_i$ since $varphi_i(x) = frac{psi_i(x)}{psi_i(x)} = 1$ for $x$ sufficiently close to $b$.
    $endgroup$
    – Pratyush Sarkar
    Feb 17 '17 at 6:03





















5












$begingroup$

I belive that your assertion is correct. The functions $varphi_{i}$ satisfy all of the conditions of Theorem 3-11. I don't see why Spivak used such an $f$. Particularly since the support of $f$ contains $A$. If at least the support of $f$ lied in $A$ then $f=sum_{i=1}^{n}fcdotvarphi_{i}$, thus giving a representation of $f$ as a sum of functions with small supports.



Since $A$ is compact we may assume WLOG that the $U_{i}$ are bounded. Therefore, by construction, the supports of the $psi_{i}$ are compact. Hence, the word "closed" in item ($4$) of Theorem 3-11 can be changed to "compact". The proof remains unchanged. This helps clarify the first statement of the proof of Theorem 3-12.



Also, note as well that the functions $varphi_{i}$ are $C^{infty}$. This basically follows from Problem 2-26.



Posts related to the section:




  1. An application of partitions of unity: integrating over open sets and here


  2. Do we need additional assumptions for problem 3-37 (b) in Spivak´s calculus on Manifolds?


  3. Problem 3-38 in Spivak´s Calculus on Manifolds


  4. Extended integral in Spivak’s Calculus on Manifolds







share|cite|improve this answer











$endgroup$













  • $begingroup$
    Firstly, in property 4, isn't the closed set bounded? Which would mean it's compact? Secondly, in the proof it says "let $psi_i$ be non-negative $C^infty$ function positive on $D_i$ and $0$ outside some closed set contained in $U_i$". Denote the closed set as $S_i$. Then $varphi_i$ has the same property, namely there is the set $U_i$ such that $varphi_i = 0$ outside $S_i$ contained in $U_i$. So isn't property 4 already fulfilled?
    $endgroup$
    – Pratyush Sarkar
    Aug 24 '13 at 15:16












  • $begingroup$
    @Pratyush: I have looked at this more carefully and updated my answer.
    $endgroup$
    – John
    Aug 24 '13 at 21:10










  • $begingroup$
    Thanks for your help. I read the theorem and proof multiple times before and I couldn't figure out the purpose of the function $f$. I thought I was missing something really obvious.
    $endgroup$
    – Pratyush Sarkar
    Aug 24 '13 at 22:51



















1












$begingroup$

Merry Christmas ! I am sorry to bother you.I revised my answer.Looking foward to your comments.



First of all, as Mr.John mentioned in his answer,the functions $varphi_{i}$ have already satisfied all of the conditions of Th 3-11 but except for $textit{(4)}$. A crucial question in Spivak's proof is: why the author "redundantly" required $fcdotvarphi_{i}$ rather than $varphi_{i}$ ?
In my way of thinking , in accord with problem 2-26$^*$(d),the author wanted to extend $varphi_{i}$ from its domain$- $an open subset in $mathbf {R}^{n}$$- $to the whole of $mathbf {R}^{n}$ smoothly and then guarantee such those extended functions $fcdot varphi_{i}$ satisfying all of four conditions.



The following is details:



Def. $;phi:X rightarrow mathbf {R}$ is a continuous real-valued function whose domain is an arbitrary set $X$ in $mathbf {R}^{n}$, the support of $phi$ is defined as the closure of the subset of $X$ where $phi$ is non-zero $i.e.,:$$supp(phi):=$the closure of the set $left{mathbf{x} in X: phi(mathbf{x})ne0 right }.$



Def. $;phiin C^{infty}(A,B)$ denotes $phi:(mathbf {R}^{n}supset )Arightarrow B(subsetmathbf {R}^{m})$ is a $C^{infty}$ function.



From the case 1,the compact sets $D_{i}(i=1,cdots,n)$ whose interiors cover $A$.In order to explain why the author modified $varphi_{i}$ with $fcdot varphi_{i}$ briefly,we can choose a specific open subset$- U:=bigcup_{i=1}^{n}int D_{i}$$- $to set forth.If in the simplest case we need $varphi_{i}$ multiplied by $f$,not to mention in most cases.



$textbf{1.}$



$psi_iin C^{infty}(U_i,mathbf{R}),$ which is positive on $D_i$ and $0$ outside of some closed set contained in $U_i$ .(problem 2-26$^*$(d))



Define $$widetilde psi_{i}:=left{begin{matrix}
psi_{i}& xin U_i\
0& xin {U_i}^{c}
end{matrix}right.quad (i=1,2,cdots,n),$$
then we have $widetilde psi_{i}in C^{infty}(mathbf{R}^{n},mathbf{R})$,$;widetilde psi_{i}bigg|_{U_{i}}=widetilde psi_{i}$ and $psi_{i}$ (each domain is $U_{i}$) can be smoothly extended to $widetilde psi_{i}$(each domian is $mathbf{R}^{n}$).



All functions $$varphi_{i}=frac{psi_{i}}{sum_{k=1}^{n}psi_{k}};(i=1,cdots,n)$$ are only constructed on $U$. $$ forall : xin U,: sum_{k=1}^{n}widetildepsi_{k}ne 0 ;;widetildepsi_{k}in C^{infty}(mathbf{R}^{n},mathbf{R}).$$ $$Longrightarrow varphi_{i}=frac{psi_{i}}{sum_{k=1}^{n}psi_{k}}=frac{widetildepsi_{i}}{sum_{k=1}^{n}widetildepsi_{k}}in C^{infty}(U,mathbf{R}) quad (i=1,2,cdots,n).$$



$textbf{2.}$



$fin C^{infty}(U,mathbf{R}),$ which value is $1$ on $A$ and $0$ outside of some closed set contained in $U$.(problem 2-26$^*$(d))



$\$Obviously,we have $fcdot varphi_{i}in C^{infty}(U,mathbf{R});(i=1,2,cdots,n).$ Note that each $fcdot varphi_{i}in C^{infty}(U,mathbf{R})$ is only constructed on $U:!$



$textbf{3.}$



Finally,we can extend $fcdot varphi_{i}$ from $U$ to the whole of $mathbf {R}^{n}$ smoothly.



In fact,since $supp(fcdotvarphi_{i})subset U$,
define $$widetilde{fcdot varphi_{i}}:=left{begin{matrix}
fcdot varphi_{i}& xin supp(fcdotvarphi_{i})\
0 & xnotin supp(fcdotvarphi_{i})
end{matrix}right.quad (i=1,2,cdots,n),$$
then we have $widetilde {fcdot varphi_{i}}in C^{infty}(mathbf{R}^{n},mathbf{R})$,$;widetilde {fcdot varphi_{i}}bigg|_{U}=fcdotvarphi_{i}$ and $fcdot varphi_{i}$ (each domain is $U$) can be smoothly extended to $widetilde {fcdot varphi_{i}}$ (each domian is $mathbf{R}^{n}).qquadqquadqquadqquadqquadqquadqquadqquadblacksquare$





A schematic diagram
(A schematic diagram concretising above elaboration)






share|cite|improve this answer











$endgroup$













  • $begingroup$
    Thanks for elaborating, this makes sense. A few comments: 1. Actually the functions $varphi_i$ do not satisfy property 4 of the theorem which is why we need $f$ to kill it off near some parts of the boundary of $U_i$. 2. In your method, $U$ is defined slightly differently but the result is the same, support of $widetilde{fcdot varphi_{i}}$ is contained inside $U_i$ as required by property 4. I want to point out that this can't be made stronger to support contained in $D_i$. So I think I prefer the $U$ as defined by Spivak because it is easier to see the containment in $U_i$.
    $endgroup$
    – Pratyush Sarkar
    Dec 26 '18 at 23:06











Your Answer





StackExchange.ifUsing("editor", function () {
return StackExchange.using("mathjaxEditing", function () {
StackExchange.MarkdownEditor.creationCallbacks.add(function (editor, postfix) {
StackExchange.mathjaxEditing.prepareWmdForMathJax(editor, postfix, [["$", "$"], ["\\(","\\)"]]);
});
});
}, "mathjax-editing");

StackExchange.ready(function() {
var channelOptions = {
tags: "".split(" "),
id: "69"
};
initTagRenderer("".split(" "), "".split(" "), channelOptions);

StackExchange.using("externalEditor", function() {
// Have to fire editor after snippets, if snippets enabled
if (StackExchange.settings.snippets.snippetsEnabled) {
StackExchange.using("snippets", function() {
createEditor();
});
}
else {
createEditor();
}
});

function createEditor() {
StackExchange.prepareEditor({
heartbeatType: 'answer',
autoActivateHeartbeat: false,
convertImagesToLinks: true,
noModals: true,
showLowRepImageUploadWarning: true,
reputationToPostImages: 10,
bindNavPrevention: true,
postfix: "",
imageUploader: {
brandingHtml: "Powered by u003ca class="icon-imgur-white" href="https://imgur.com/"u003eu003c/au003e",
contentPolicyHtml: "User contributions licensed under u003ca href="https://creativecommons.org/licenses/by-sa/3.0/"u003ecc by-sa 3.0 with attribution requiredu003c/au003e u003ca href="https://stackoverflow.com/legal/content-policy"u003e(content policy)u003c/au003e",
allowUrls: true
},
noCode: true, onDemand: true,
discardSelector: ".discard-answer"
,immediatelyShowMarkdownHelp:true
});


}
});














draft saved

draft discarded


















StackExchange.ready(
function () {
StackExchange.openid.initPostLogin('.new-post-login', 'https%3a%2f%2fmath.stackexchange.com%2fquestions%2f475060%2fpartition-of-unity-in-spivaks-calculus-on-manifolds%23new-answer', 'question_page');
}
);

Post as a guest















Required, but never shown

























3 Answers
3






active

oldest

votes








3 Answers
3






active

oldest

votes









active

oldest

votes






active

oldest

votes









6












$begingroup$

I was looking back at my question today for some reason and immediately saw why the function $f$ is required. Although $psi_i$ is smooth with compact support in $U_i$, the functions $varphi_i$ can only be defined on $U$ where $sum_{i = 1}^npsi_i > 0$. The problem is that $varphi_i$ usually does not go to zero at the boundary of $operatorname{supp}(psi_i)$ (much less smoothly extend to zero outside the boundary). You can see this near the boundary of a $operatorname{supp}(psi_i)$ which is away from all other $operatorname{supp}(psi_j)$. Near this boundary $varphi_i(x) = frac{psi_i(x)}{psi_i(x)} = 1$. The solution is to use a cutoff function $f$ which forces everything to smoothly go to $0$ near the boundary of $U$.






share|cite|improve this answer











$endgroup$













  • $begingroup$
    I think you mean $varphi_{i}$ (not $psi_{i}$) does not usually go to zero at the boundary of $text{supp}(psi_{i})$.
    $endgroup$
    – fourierwho
    May 31 '16 at 16:37










  • $begingroup$
    @fourierwho Yes. Thanks for the correction.
    $endgroup$
    – Pratyush Sarkar
    Jul 25 '16 at 23:14










  • $begingroup$
    @PratyushSarkar, the fact of $phi_i$ doesn't smoothly extend to zero outside the boundary of $operatorname{supp} (psi_i)$ doesn't contradicts the fact of $psi_i$ are $C^{infty}$ by the way was built? If your answer is not, so I think that I didn't understand the purpose of $f$.
    $endgroup$
    – George
    Jan 29 '17 at 18:28












  • $begingroup$
    @George No there is no contradiction. What I explained is, although $psi_i$ obviously extends to $0$ smoothly outside $operatorname{supp} (psi_i)$, the biggest set where we can define $varphi_i$ is only on $U = bigcup_{i = 1}^n operatorname{supp} (psi_i)$ and it is possible that near the boundary of $U$ which, say, coincides with $operatorname{supp} (psi_i)$ and is away from all other $operatorname{supp} (psi_j)$, the value of $varphi_i(x)$ tends to $1$.
    $endgroup$
    – Pratyush Sarkar
    Feb 17 '17 at 5:56












  • $begingroup$
    More explicitly, we can easily think of (and draw) an example where $B_i = partial U setminus left(bigcup_{j neq i} operatorname{supp} (psi_j)right)$ is nonempty and in that case we have $lim_{x to b} varphi_i(x) = 1$ for any $b in B_i$ since $varphi_i(x) = frac{psi_i(x)}{psi_i(x)} = 1$ for $x$ sufficiently close to $b$.
    $endgroup$
    – Pratyush Sarkar
    Feb 17 '17 at 6:03


















6












$begingroup$

I was looking back at my question today for some reason and immediately saw why the function $f$ is required. Although $psi_i$ is smooth with compact support in $U_i$, the functions $varphi_i$ can only be defined on $U$ where $sum_{i = 1}^npsi_i > 0$. The problem is that $varphi_i$ usually does not go to zero at the boundary of $operatorname{supp}(psi_i)$ (much less smoothly extend to zero outside the boundary). You can see this near the boundary of a $operatorname{supp}(psi_i)$ which is away from all other $operatorname{supp}(psi_j)$. Near this boundary $varphi_i(x) = frac{psi_i(x)}{psi_i(x)} = 1$. The solution is to use a cutoff function $f$ which forces everything to smoothly go to $0$ near the boundary of $U$.






share|cite|improve this answer











$endgroup$













  • $begingroup$
    I think you mean $varphi_{i}$ (not $psi_{i}$) does not usually go to zero at the boundary of $text{supp}(psi_{i})$.
    $endgroup$
    – fourierwho
    May 31 '16 at 16:37










  • $begingroup$
    @fourierwho Yes. Thanks for the correction.
    $endgroup$
    – Pratyush Sarkar
    Jul 25 '16 at 23:14










  • $begingroup$
    @PratyushSarkar, the fact of $phi_i$ doesn't smoothly extend to zero outside the boundary of $operatorname{supp} (psi_i)$ doesn't contradicts the fact of $psi_i$ are $C^{infty}$ by the way was built? If your answer is not, so I think that I didn't understand the purpose of $f$.
    $endgroup$
    – George
    Jan 29 '17 at 18:28












  • $begingroup$
    @George No there is no contradiction. What I explained is, although $psi_i$ obviously extends to $0$ smoothly outside $operatorname{supp} (psi_i)$, the biggest set where we can define $varphi_i$ is only on $U = bigcup_{i = 1}^n operatorname{supp} (psi_i)$ and it is possible that near the boundary of $U$ which, say, coincides with $operatorname{supp} (psi_i)$ and is away from all other $operatorname{supp} (psi_j)$, the value of $varphi_i(x)$ tends to $1$.
    $endgroup$
    – Pratyush Sarkar
    Feb 17 '17 at 5:56












  • $begingroup$
    More explicitly, we can easily think of (and draw) an example where $B_i = partial U setminus left(bigcup_{j neq i} operatorname{supp} (psi_j)right)$ is nonempty and in that case we have $lim_{x to b} varphi_i(x) = 1$ for any $b in B_i$ since $varphi_i(x) = frac{psi_i(x)}{psi_i(x)} = 1$ for $x$ sufficiently close to $b$.
    $endgroup$
    – Pratyush Sarkar
    Feb 17 '17 at 6:03
















6












6








6





$begingroup$

I was looking back at my question today for some reason and immediately saw why the function $f$ is required. Although $psi_i$ is smooth with compact support in $U_i$, the functions $varphi_i$ can only be defined on $U$ where $sum_{i = 1}^npsi_i > 0$. The problem is that $varphi_i$ usually does not go to zero at the boundary of $operatorname{supp}(psi_i)$ (much less smoothly extend to zero outside the boundary). You can see this near the boundary of a $operatorname{supp}(psi_i)$ which is away from all other $operatorname{supp}(psi_j)$. Near this boundary $varphi_i(x) = frac{psi_i(x)}{psi_i(x)} = 1$. The solution is to use a cutoff function $f$ which forces everything to smoothly go to $0$ near the boundary of $U$.






share|cite|improve this answer











$endgroup$



I was looking back at my question today for some reason and immediately saw why the function $f$ is required. Although $psi_i$ is smooth with compact support in $U_i$, the functions $varphi_i$ can only be defined on $U$ where $sum_{i = 1}^npsi_i > 0$. The problem is that $varphi_i$ usually does not go to zero at the boundary of $operatorname{supp}(psi_i)$ (much less smoothly extend to zero outside the boundary). You can see this near the boundary of a $operatorname{supp}(psi_i)$ which is away from all other $operatorname{supp}(psi_j)$. Near this boundary $varphi_i(x) = frac{psi_i(x)}{psi_i(x)} = 1$. The solution is to use a cutoff function $f$ which forces everything to smoothly go to $0$ near the boundary of $U$.







share|cite|improve this answer














share|cite|improve this answer



share|cite|improve this answer








edited Feb 17 '17 at 5:30

























answered Jan 27 '16 at 5:20









Pratyush SarkarPratyush Sarkar

2,7801127




2,7801127












  • $begingroup$
    I think you mean $varphi_{i}$ (not $psi_{i}$) does not usually go to zero at the boundary of $text{supp}(psi_{i})$.
    $endgroup$
    – fourierwho
    May 31 '16 at 16:37










  • $begingroup$
    @fourierwho Yes. Thanks for the correction.
    $endgroup$
    – Pratyush Sarkar
    Jul 25 '16 at 23:14










  • $begingroup$
    @PratyushSarkar, the fact of $phi_i$ doesn't smoothly extend to zero outside the boundary of $operatorname{supp} (psi_i)$ doesn't contradicts the fact of $psi_i$ are $C^{infty}$ by the way was built? If your answer is not, so I think that I didn't understand the purpose of $f$.
    $endgroup$
    – George
    Jan 29 '17 at 18:28












  • $begingroup$
    @George No there is no contradiction. What I explained is, although $psi_i$ obviously extends to $0$ smoothly outside $operatorname{supp} (psi_i)$, the biggest set where we can define $varphi_i$ is only on $U = bigcup_{i = 1}^n operatorname{supp} (psi_i)$ and it is possible that near the boundary of $U$ which, say, coincides with $operatorname{supp} (psi_i)$ and is away from all other $operatorname{supp} (psi_j)$, the value of $varphi_i(x)$ tends to $1$.
    $endgroup$
    – Pratyush Sarkar
    Feb 17 '17 at 5:56












  • $begingroup$
    More explicitly, we can easily think of (and draw) an example where $B_i = partial U setminus left(bigcup_{j neq i} operatorname{supp} (psi_j)right)$ is nonempty and in that case we have $lim_{x to b} varphi_i(x) = 1$ for any $b in B_i$ since $varphi_i(x) = frac{psi_i(x)}{psi_i(x)} = 1$ for $x$ sufficiently close to $b$.
    $endgroup$
    – Pratyush Sarkar
    Feb 17 '17 at 6:03




















  • $begingroup$
    I think you mean $varphi_{i}$ (not $psi_{i}$) does not usually go to zero at the boundary of $text{supp}(psi_{i})$.
    $endgroup$
    – fourierwho
    May 31 '16 at 16:37










  • $begingroup$
    @fourierwho Yes. Thanks for the correction.
    $endgroup$
    – Pratyush Sarkar
    Jul 25 '16 at 23:14










  • $begingroup$
    @PratyushSarkar, the fact of $phi_i$ doesn't smoothly extend to zero outside the boundary of $operatorname{supp} (psi_i)$ doesn't contradicts the fact of $psi_i$ are $C^{infty}$ by the way was built? If your answer is not, so I think that I didn't understand the purpose of $f$.
    $endgroup$
    – George
    Jan 29 '17 at 18:28












  • $begingroup$
    @George No there is no contradiction. What I explained is, although $psi_i$ obviously extends to $0$ smoothly outside $operatorname{supp} (psi_i)$, the biggest set where we can define $varphi_i$ is only on $U = bigcup_{i = 1}^n operatorname{supp} (psi_i)$ and it is possible that near the boundary of $U$ which, say, coincides with $operatorname{supp} (psi_i)$ and is away from all other $operatorname{supp} (psi_j)$, the value of $varphi_i(x)$ tends to $1$.
    $endgroup$
    – Pratyush Sarkar
    Feb 17 '17 at 5:56












  • $begingroup$
    More explicitly, we can easily think of (and draw) an example where $B_i = partial U setminus left(bigcup_{j neq i} operatorname{supp} (psi_j)right)$ is nonempty and in that case we have $lim_{x to b} varphi_i(x) = 1$ for any $b in B_i$ since $varphi_i(x) = frac{psi_i(x)}{psi_i(x)} = 1$ for $x$ sufficiently close to $b$.
    $endgroup$
    – Pratyush Sarkar
    Feb 17 '17 at 6:03


















$begingroup$
I think you mean $varphi_{i}$ (not $psi_{i}$) does not usually go to zero at the boundary of $text{supp}(psi_{i})$.
$endgroup$
– fourierwho
May 31 '16 at 16:37




$begingroup$
I think you mean $varphi_{i}$ (not $psi_{i}$) does not usually go to zero at the boundary of $text{supp}(psi_{i})$.
$endgroup$
– fourierwho
May 31 '16 at 16:37












$begingroup$
@fourierwho Yes. Thanks for the correction.
$endgroup$
– Pratyush Sarkar
Jul 25 '16 at 23:14




$begingroup$
@fourierwho Yes. Thanks for the correction.
$endgroup$
– Pratyush Sarkar
Jul 25 '16 at 23:14












$begingroup$
@PratyushSarkar, the fact of $phi_i$ doesn't smoothly extend to zero outside the boundary of $operatorname{supp} (psi_i)$ doesn't contradicts the fact of $psi_i$ are $C^{infty}$ by the way was built? If your answer is not, so I think that I didn't understand the purpose of $f$.
$endgroup$
– George
Jan 29 '17 at 18:28






$begingroup$
@PratyushSarkar, the fact of $phi_i$ doesn't smoothly extend to zero outside the boundary of $operatorname{supp} (psi_i)$ doesn't contradicts the fact of $psi_i$ are $C^{infty}$ by the way was built? If your answer is not, so I think that I didn't understand the purpose of $f$.
$endgroup$
– George
Jan 29 '17 at 18:28














$begingroup$
@George No there is no contradiction. What I explained is, although $psi_i$ obviously extends to $0$ smoothly outside $operatorname{supp} (psi_i)$, the biggest set where we can define $varphi_i$ is only on $U = bigcup_{i = 1}^n operatorname{supp} (psi_i)$ and it is possible that near the boundary of $U$ which, say, coincides with $operatorname{supp} (psi_i)$ and is away from all other $operatorname{supp} (psi_j)$, the value of $varphi_i(x)$ tends to $1$.
$endgroup$
– Pratyush Sarkar
Feb 17 '17 at 5:56






$begingroup$
@George No there is no contradiction. What I explained is, although $psi_i$ obviously extends to $0$ smoothly outside $operatorname{supp} (psi_i)$, the biggest set where we can define $varphi_i$ is only on $U = bigcup_{i = 1}^n operatorname{supp} (psi_i)$ and it is possible that near the boundary of $U$ which, say, coincides with $operatorname{supp} (psi_i)$ and is away from all other $operatorname{supp} (psi_j)$, the value of $varphi_i(x)$ tends to $1$.
$endgroup$
– Pratyush Sarkar
Feb 17 '17 at 5:56














$begingroup$
More explicitly, we can easily think of (and draw) an example where $B_i = partial U setminus left(bigcup_{j neq i} operatorname{supp} (psi_j)right)$ is nonempty and in that case we have $lim_{x to b} varphi_i(x) = 1$ for any $b in B_i$ since $varphi_i(x) = frac{psi_i(x)}{psi_i(x)} = 1$ for $x$ sufficiently close to $b$.
$endgroup$
– Pratyush Sarkar
Feb 17 '17 at 6:03






$begingroup$
More explicitly, we can easily think of (and draw) an example where $B_i = partial U setminus left(bigcup_{j neq i} operatorname{supp} (psi_j)right)$ is nonempty and in that case we have $lim_{x to b} varphi_i(x) = 1$ for any $b in B_i$ since $varphi_i(x) = frac{psi_i(x)}{psi_i(x)} = 1$ for $x$ sufficiently close to $b$.
$endgroup$
– Pratyush Sarkar
Feb 17 '17 at 6:03













5












$begingroup$

I belive that your assertion is correct. The functions $varphi_{i}$ satisfy all of the conditions of Theorem 3-11. I don't see why Spivak used such an $f$. Particularly since the support of $f$ contains $A$. If at least the support of $f$ lied in $A$ then $f=sum_{i=1}^{n}fcdotvarphi_{i}$, thus giving a representation of $f$ as a sum of functions with small supports.



Since $A$ is compact we may assume WLOG that the $U_{i}$ are bounded. Therefore, by construction, the supports of the $psi_{i}$ are compact. Hence, the word "closed" in item ($4$) of Theorem 3-11 can be changed to "compact". The proof remains unchanged. This helps clarify the first statement of the proof of Theorem 3-12.



Also, note as well that the functions $varphi_{i}$ are $C^{infty}$. This basically follows from Problem 2-26.



Posts related to the section:




  1. An application of partitions of unity: integrating over open sets and here


  2. Do we need additional assumptions for problem 3-37 (b) in Spivak´s calculus on Manifolds?


  3. Problem 3-38 in Spivak´s Calculus on Manifolds


  4. Extended integral in Spivak’s Calculus on Manifolds







share|cite|improve this answer











$endgroup$













  • $begingroup$
    Firstly, in property 4, isn't the closed set bounded? Which would mean it's compact? Secondly, in the proof it says "let $psi_i$ be non-negative $C^infty$ function positive on $D_i$ and $0$ outside some closed set contained in $U_i$". Denote the closed set as $S_i$. Then $varphi_i$ has the same property, namely there is the set $U_i$ such that $varphi_i = 0$ outside $S_i$ contained in $U_i$. So isn't property 4 already fulfilled?
    $endgroup$
    – Pratyush Sarkar
    Aug 24 '13 at 15:16












  • $begingroup$
    @Pratyush: I have looked at this more carefully and updated my answer.
    $endgroup$
    – John
    Aug 24 '13 at 21:10










  • $begingroup$
    Thanks for your help. I read the theorem and proof multiple times before and I couldn't figure out the purpose of the function $f$. I thought I was missing something really obvious.
    $endgroup$
    – Pratyush Sarkar
    Aug 24 '13 at 22:51
















5












$begingroup$

I belive that your assertion is correct. The functions $varphi_{i}$ satisfy all of the conditions of Theorem 3-11. I don't see why Spivak used such an $f$. Particularly since the support of $f$ contains $A$. If at least the support of $f$ lied in $A$ then $f=sum_{i=1}^{n}fcdotvarphi_{i}$, thus giving a representation of $f$ as a sum of functions with small supports.



Since $A$ is compact we may assume WLOG that the $U_{i}$ are bounded. Therefore, by construction, the supports of the $psi_{i}$ are compact. Hence, the word "closed" in item ($4$) of Theorem 3-11 can be changed to "compact". The proof remains unchanged. This helps clarify the first statement of the proof of Theorem 3-12.



Also, note as well that the functions $varphi_{i}$ are $C^{infty}$. This basically follows from Problem 2-26.



Posts related to the section:




  1. An application of partitions of unity: integrating over open sets and here


  2. Do we need additional assumptions for problem 3-37 (b) in Spivak´s calculus on Manifolds?


  3. Problem 3-38 in Spivak´s Calculus on Manifolds


  4. Extended integral in Spivak’s Calculus on Manifolds







share|cite|improve this answer











$endgroup$













  • $begingroup$
    Firstly, in property 4, isn't the closed set bounded? Which would mean it's compact? Secondly, in the proof it says "let $psi_i$ be non-negative $C^infty$ function positive on $D_i$ and $0$ outside some closed set contained in $U_i$". Denote the closed set as $S_i$. Then $varphi_i$ has the same property, namely there is the set $U_i$ such that $varphi_i = 0$ outside $S_i$ contained in $U_i$. So isn't property 4 already fulfilled?
    $endgroup$
    – Pratyush Sarkar
    Aug 24 '13 at 15:16












  • $begingroup$
    @Pratyush: I have looked at this more carefully and updated my answer.
    $endgroup$
    – John
    Aug 24 '13 at 21:10










  • $begingroup$
    Thanks for your help. I read the theorem and proof multiple times before and I couldn't figure out the purpose of the function $f$. I thought I was missing something really obvious.
    $endgroup$
    – Pratyush Sarkar
    Aug 24 '13 at 22:51














5












5








5





$begingroup$

I belive that your assertion is correct. The functions $varphi_{i}$ satisfy all of the conditions of Theorem 3-11. I don't see why Spivak used such an $f$. Particularly since the support of $f$ contains $A$. If at least the support of $f$ lied in $A$ then $f=sum_{i=1}^{n}fcdotvarphi_{i}$, thus giving a representation of $f$ as a sum of functions with small supports.



Since $A$ is compact we may assume WLOG that the $U_{i}$ are bounded. Therefore, by construction, the supports of the $psi_{i}$ are compact. Hence, the word "closed" in item ($4$) of Theorem 3-11 can be changed to "compact". The proof remains unchanged. This helps clarify the first statement of the proof of Theorem 3-12.



Also, note as well that the functions $varphi_{i}$ are $C^{infty}$. This basically follows from Problem 2-26.



Posts related to the section:




  1. An application of partitions of unity: integrating over open sets and here


  2. Do we need additional assumptions for problem 3-37 (b) in Spivak´s calculus on Manifolds?


  3. Problem 3-38 in Spivak´s Calculus on Manifolds


  4. Extended integral in Spivak’s Calculus on Manifolds







share|cite|improve this answer











$endgroup$



I belive that your assertion is correct. The functions $varphi_{i}$ satisfy all of the conditions of Theorem 3-11. I don't see why Spivak used such an $f$. Particularly since the support of $f$ contains $A$. If at least the support of $f$ lied in $A$ then $f=sum_{i=1}^{n}fcdotvarphi_{i}$, thus giving a representation of $f$ as a sum of functions with small supports.



Since $A$ is compact we may assume WLOG that the $U_{i}$ are bounded. Therefore, by construction, the supports of the $psi_{i}$ are compact. Hence, the word "closed" in item ($4$) of Theorem 3-11 can be changed to "compact". The proof remains unchanged. This helps clarify the first statement of the proof of Theorem 3-12.



Also, note as well that the functions $varphi_{i}$ are $C^{infty}$. This basically follows from Problem 2-26.



Posts related to the section:




  1. An application of partitions of unity: integrating over open sets and here


  2. Do we need additional assumptions for problem 3-37 (b) in Spivak´s calculus on Manifolds?


  3. Problem 3-38 in Spivak´s Calculus on Manifolds


  4. Extended integral in Spivak’s Calculus on Manifolds








share|cite|improve this answer














share|cite|improve this answer



share|cite|improve this answer








edited Apr 13 '17 at 12:58









Community

1




1










answered Aug 24 '13 at 14:28









JohnJohn

1,533924




1,533924












  • $begingroup$
    Firstly, in property 4, isn't the closed set bounded? Which would mean it's compact? Secondly, in the proof it says "let $psi_i$ be non-negative $C^infty$ function positive on $D_i$ and $0$ outside some closed set contained in $U_i$". Denote the closed set as $S_i$. Then $varphi_i$ has the same property, namely there is the set $U_i$ such that $varphi_i = 0$ outside $S_i$ contained in $U_i$. So isn't property 4 already fulfilled?
    $endgroup$
    – Pratyush Sarkar
    Aug 24 '13 at 15:16












  • $begingroup$
    @Pratyush: I have looked at this more carefully and updated my answer.
    $endgroup$
    – John
    Aug 24 '13 at 21:10










  • $begingroup$
    Thanks for your help. I read the theorem and proof multiple times before and I couldn't figure out the purpose of the function $f$. I thought I was missing something really obvious.
    $endgroup$
    – Pratyush Sarkar
    Aug 24 '13 at 22:51


















  • $begingroup$
    Firstly, in property 4, isn't the closed set bounded? Which would mean it's compact? Secondly, in the proof it says "let $psi_i$ be non-negative $C^infty$ function positive on $D_i$ and $0$ outside some closed set contained in $U_i$". Denote the closed set as $S_i$. Then $varphi_i$ has the same property, namely there is the set $U_i$ such that $varphi_i = 0$ outside $S_i$ contained in $U_i$. So isn't property 4 already fulfilled?
    $endgroup$
    – Pratyush Sarkar
    Aug 24 '13 at 15:16












  • $begingroup$
    @Pratyush: I have looked at this more carefully and updated my answer.
    $endgroup$
    – John
    Aug 24 '13 at 21:10










  • $begingroup$
    Thanks for your help. I read the theorem and proof multiple times before and I couldn't figure out the purpose of the function $f$. I thought I was missing something really obvious.
    $endgroup$
    – Pratyush Sarkar
    Aug 24 '13 at 22:51
















$begingroup$
Firstly, in property 4, isn't the closed set bounded? Which would mean it's compact? Secondly, in the proof it says "let $psi_i$ be non-negative $C^infty$ function positive on $D_i$ and $0$ outside some closed set contained in $U_i$". Denote the closed set as $S_i$. Then $varphi_i$ has the same property, namely there is the set $U_i$ such that $varphi_i = 0$ outside $S_i$ contained in $U_i$. So isn't property 4 already fulfilled?
$endgroup$
– Pratyush Sarkar
Aug 24 '13 at 15:16






$begingroup$
Firstly, in property 4, isn't the closed set bounded? Which would mean it's compact? Secondly, in the proof it says "let $psi_i$ be non-negative $C^infty$ function positive on $D_i$ and $0$ outside some closed set contained in $U_i$". Denote the closed set as $S_i$. Then $varphi_i$ has the same property, namely there is the set $U_i$ such that $varphi_i = 0$ outside $S_i$ contained in $U_i$. So isn't property 4 already fulfilled?
$endgroup$
– Pratyush Sarkar
Aug 24 '13 at 15:16














$begingroup$
@Pratyush: I have looked at this more carefully and updated my answer.
$endgroup$
– John
Aug 24 '13 at 21:10




$begingroup$
@Pratyush: I have looked at this more carefully and updated my answer.
$endgroup$
– John
Aug 24 '13 at 21:10












$begingroup$
Thanks for your help. I read the theorem and proof multiple times before and I couldn't figure out the purpose of the function $f$. I thought I was missing something really obvious.
$endgroup$
– Pratyush Sarkar
Aug 24 '13 at 22:51




$begingroup$
Thanks for your help. I read the theorem and proof multiple times before and I couldn't figure out the purpose of the function $f$. I thought I was missing something really obvious.
$endgroup$
– Pratyush Sarkar
Aug 24 '13 at 22:51











1












$begingroup$

Merry Christmas ! I am sorry to bother you.I revised my answer.Looking foward to your comments.



First of all, as Mr.John mentioned in his answer,the functions $varphi_{i}$ have already satisfied all of the conditions of Th 3-11 but except for $textit{(4)}$. A crucial question in Spivak's proof is: why the author "redundantly" required $fcdotvarphi_{i}$ rather than $varphi_{i}$ ?
In my way of thinking , in accord with problem 2-26$^*$(d),the author wanted to extend $varphi_{i}$ from its domain$- $an open subset in $mathbf {R}^{n}$$- $to the whole of $mathbf {R}^{n}$ smoothly and then guarantee such those extended functions $fcdot varphi_{i}$ satisfying all of four conditions.



The following is details:



Def. $;phi:X rightarrow mathbf {R}$ is a continuous real-valued function whose domain is an arbitrary set $X$ in $mathbf {R}^{n}$, the support of $phi$ is defined as the closure of the subset of $X$ where $phi$ is non-zero $i.e.,:$$supp(phi):=$the closure of the set $left{mathbf{x} in X: phi(mathbf{x})ne0 right }.$



Def. $;phiin C^{infty}(A,B)$ denotes $phi:(mathbf {R}^{n}supset )Arightarrow B(subsetmathbf {R}^{m})$ is a $C^{infty}$ function.



From the case 1,the compact sets $D_{i}(i=1,cdots,n)$ whose interiors cover $A$.In order to explain why the author modified $varphi_{i}$ with $fcdot varphi_{i}$ briefly,we can choose a specific open subset$- U:=bigcup_{i=1}^{n}int D_{i}$$- $to set forth.If in the simplest case we need $varphi_{i}$ multiplied by $f$,not to mention in most cases.



$textbf{1.}$



$psi_iin C^{infty}(U_i,mathbf{R}),$ which is positive on $D_i$ and $0$ outside of some closed set contained in $U_i$ .(problem 2-26$^*$(d))



Define $$widetilde psi_{i}:=left{begin{matrix}
psi_{i}& xin U_i\
0& xin {U_i}^{c}
end{matrix}right.quad (i=1,2,cdots,n),$$
then we have $widetilde psi_{i}in C^{infty}(mathbf{R}^{n},mathbf{R})$,$;widetilde psi_{i}bigg|_{U_{i}}=widetilde psi_{i}$ and $psi_{i}$ (each domain is $U_{i}$) can be smoothly extended to $widetilde psi_{i}$(each domian is $mathbf{R}^{n}$).



All functions $$varphi_{i}=frac{psi_{i}}{sum_{k=1}^{n}psi_{k}};(i=1,cdots,n)$$ are only constructed on $U$. $$ forall : xin U,: sum_{k=1}^{n}widetildepsi_{k}ne 0 ;;widetildepsi_{k}in C^{infty}(mathbf{R}^{n},mathbf{R}).$$ $$Longrightarrow varphi_{i}=frac{psi_{i}}{sum_{k=1}^{n}psi_{k}}=frac{widetildepsi_{i}}{sum_{k=1}^{n}widetildepsi_{k}}in C^{infty}(U,mathbf{R}) quad (i=1,2,cdots,n).$$



$textbf{2.}$



$fin C^{infty}(U,mathbf{R}),$ which value is $1$ on $A$ and $0$ outside of some closed set contained in $U$.(problem 2-26$^*$(d))



$\$Obviously,we have $fcdot varphi_{i}in C^{infty}(U,mathbf{R});(i=1,2,cdots,n).$ Note that each $fcdot varphi_{i}in C^{infty}(U,mathbf{R})$ is only constructed on $U:!$



$textbf{3.}$



Finally,we can extend $fcdot varphi_{i}$ from $U$ to the whole of $mathbf {R}^{n}$ smoothly.



In fact,since $supp(fcdotvarphi_{i})subset U$,
define $$widetilde{fcdot varphi_{i}}:=left{begin{matrix}
fcdot varphi_{i}& xin supp(fcdotvarphi_{i})\
0 & xnotin supp(fcdotvarphi_{i})
end{matrix}right.quad (i=1,2,cdots,n),$$
then we have $widetilde {fcdot varphi_{i}}in C^{infty}(mathbf{R}^{n},mathbf{R})$,$;widetilde {fcdot varphi_{i}}bigg|_{U}=fcdotvarphi_{i}$ and $fcdot varphi_{i}$ (each domain is $U$) can be smoothly extended to $widetilde {fcdot varphi_{i}}$ (each domian is $mathbf{R}^{n}).qquadqquadqquadqquadqquadqquadqquadqquadblacksquare$





A schematic diagram
(A schematic diagram concretising above elaboration)






share|cite|improve this answer











$endgroup$













  • $begingroup$
    Thanks for elaborating, this makes sense. A few comments: 1. Actually the functions $varphi_i$ do not satisfy property 4 of the theorem which is why we need $f$ to kill it off near some parts of the boundary of $U_i$. 2. In your method, $U$ is defined slightly differently but the result is the same, support of $widetilde{fcdot varphi_{i}}$ is contained inside $U_i$ as required by property 4. I want to point out that this can't be made stronger to support contained in $D_i$. So I think I prefer the $U$ as defined by Spivak because it is easier to see the containment in $U_i$.
    $endgroup$
    – Pratyush Sarkar
    Dec 26 '18 at 23:06
















1












$begingroup$

Merry Christmas ! I am sorry to bother you.I revised my answer.Looking foward to your comments.



First of all, as Mr.John mentioned in his answer,the functions $varphi_{i}$ have already satisfied all of the conditions of Th 3-11 but except for $textit{(4)}$. A crucial question in Spivak's proof is: why the author "redundantly" required $fcdotvarphi_{i}$ rather than $varphi_{i}$ ?
In my way of thinking , in accord with problem 2-26$^*$(d),the author wanted to extend $varphi_{i}$ from its domain$- $an open subset in $mathbf {R}^{n}$$- $to the whole of $mathbf {R}^{n}$ smoothly and then guarantee such those extended functions $fcdot varphi_{i}$ satisfying all of four conditions.



The following is details:



Def. $;phi:X rightarrow mathbf {R}$ is a continuous real-valued function whose domain is an arbitrary set $X$ in $mathbf {R}^{n}$, the support of $phi$ is defined as the closure of the subset of $X$ where $phi$ is non-zero $i.e.,:$$supp(phi):=$the closure of the set $left{mathbf{x} in X: phi(mathbf{x})ne0 right }.$



Def. $;phiin C^{infty}(A,B)$ denotes $phi:(mathbf {R}^{n}supset )Arightarrow B(subsetmathbf {R}^{m})$ is a $C^{infty}$ function.



From the case 1,the compact sets $D_{i}(i=1,cdots,n)$ whose interiors cover $A$.In order to explain why the author modified $varphi_{i}$ with $fcdot varphi_{i}$ briefly,we can choose a specific open subset$- U:=bigcup_{i=1}^{n}int D_{i}$$- $to set forth.If in the simplest case we need $varphi_{i}$ multiplied by $f$,not to mention in most cases.



$textbf{1.}$



$psi_iin C^{infty}(U_i,mathbf{R}),$ which is positive on $D_i$ and $0$ outside of some closed set contained in $U_i$ .(problem 2-26$^*$(d))



Define $$widetilde psi_{i}:=left{begin{matrix}
psi_{i}& xin U_i\
0& xin {U_i}^{c}
end{matrix}right.quad (i=1,2,cdots,n),$$
then we have $widetilde psi_{i}in C^{infty}(mathbf{R}^{n},mathbf{R})$,$;widetilde psi_{i}bigg|_{U_{i}}=widetilde psi_{i}$ and $psi_{i}$ (each domain is $U_{i}$) can be smoothly extended to $widetilde psi_{i}$(each domian is $mathbf{R}^{n}$).



All functions $$varphi_{i}=frac{psi_{i}}{sum_{k=1}^{n}psi_{k}};(i=1,cdots,n)$$ are only constructed on $U$. $$ forall : xin U,: sum_{k=1}^{n}widetildepsi_{k}ne 0 ;;widetildepsi_{k}in C^{infty}(mathbf{R}^{n},mathbf{R}).$$ $$Longrightarrow varphi_{i}=frac{psi_{i}}{sum_{k=1}^{n}psi_{k}}=frac{widetildepsi_{i}}{sum_{k=1}^{n}widetildepsi_{k}}in C^{infty}(U,mathbf{R}) quad (i=1,2,cdots,n).$$



$textbf{2.}$



$fin C^{infty}(U,mathbf{R}),$ which value is $1$ on $A$ and $0$ outside of some closed set contained in $U$.(problem 2-26$^*$(d))



$\$Obviously,we have $fcdot varphi_{i}in C^{infty}(U,mathbf{R});(i=1,2,cdots,n).$ Note that each $fcdot varphi_{i}in C^{infty}(U,mathbf{R})$ is only constructed on $U:!$



$textbf{3.}$



Finally,we can extend $fcdot varphi_{i}$ from $U$ to the whole of $mathbf {R}^{n}$ smoothly.



In fact,since $supp(fcdotvarphi_{i})subset U$,
define $$widetilde{fcdot varphi_{i}}:=left{begin{matrix}
fcdot varphi_{i}& xin supp(fcdotvarphi_{i})\
0 & xnotin supp(fcdotvarphi_{i})
end{matrix}right.quad (i=1,2,cdots,n),$$
then we have $widetilde {fcdot varphi_{i}}in C^{infty}(mathbf{R}^{n},mathbf{R})$,$;widetilde {fcdot varphi_{i}}bigg|_{U}=fcdotvarphi_{i}$ and $fcdot varphi_{i}$ (each domain is $U$) can be smoothly extended to $widetilde {fcdot varphi_{i}}$ (each domian is $mathbf{R}^{n}).qquadqquadqquadqquadqquadqquadqquadqquadblacksquare$





A schematic diagram
(A schematic diagram concretising above elaboration)






share|cite|improve this answer











$endgroup$













  • $begingroup$
    Thanks for elaborating, this makes sense. A few comments: 1. Actually the functions $varphi_i$ do not satisfy property 4 of the theorem which is why we need $f$ to kill it off near some parts of the boundary of $U_i$. 2. In your method, $U$ is defined slightly differently but the result is the same, support of $widetilde{fcdot varphi_{i}}$ is contained inside $U_i$ as required by property 4. I want to point out that this can't be made stronger to support contained in $D_i$. So I think I prefer the $U$ as defined by Spivak because it is easier to see the containment in $U_i$.
    $endgroup$
    – Pratyush Sarkar
    Dec 26 '18 at 23:06














1












1








1





$begingroup$

Merry Christmas ! I am sorry to bother you.I revised my answer.Looking foward to your comments.



First of all, as Mr.John mentioned in his answer,the functions $varphi_{i}$ have already satisfied all of the conditions of Th 3-11 but except for $textit{(4)}$. A crucial question in Spivak's proof is: why the author "redundantly" required $fcdotvarphi_{i}$ rather than $varphi_{i}$ ?
In my way of thinking , in accord with problem 2-26$^*$(d),the author wanted to extend $varphi_{i}$ from its domain$- $an open subset in $mathbf {R}^{n}$$- $to the whole of $mathbf {R}^{n}$ smoothly and then guarantee such those extended functions $fcdot varphi_{i}$ satisfying all of four conditions.



The following is details:



Def. $;phi:X rightarrow mathbf {R}$ is a continuous real-valued function whose domain is an arbitrary set $X$ in $mathbf {R}^{n}$, the support of $phi$ is defined as the closure of the subset of $X$ where $phi$ is non-zero $i.e.,:$$supp(phi):=$the closure of the set $left{mathbf{x} in X: phi(mathbf{x})ne0 right }.$



Def. $;phiin C^{infty}(A,B)$ denotes $phi:(mathbf {R}^{n}supset )Arightarrow B(subsetmathbf {R}^{m})$ is a $C^{infty}$ function.



From the case 1,the compact sets $D_{i}(i=1,cdots,n)$ whose interiors cover $A$.In order to explain why the author modified $varphi_{i}$ with $fcdot varphi_{i}$ briefly,we can choose a specific open subset$- U:=bigcup_{i=1}^{n}int D_{i}$$- $to set forth.If in the simplest case we need $varphi_{i}$ multiplied by $f$,not to mention in most cases.



$textbf{1.}$



$psi_iin C^{infty}(U_i,mathbf{R}),$ which is positive on $D_i$ and $0$ outside of some closed set contained in $U_i$ .(problem 2-26$^*$(d))



Define $$widetilde psi_{i}:=left{begin{matrix}
psi_{i}& xin U_i\
0& xin {U_i}^{c}
end{matrix}right.quad (i=1,2,cdots,n),$$
then we have $widetilde psi_{i}in C^{infty}(mathbf{R}^{n},mathbf{R})$,$;widetilde psi_{i}bigg|_{U_{i}}=widetilde psi_{i}$ and $psi_{i}$ (each domain is $U_{i}$) can be smoothly extended to $widetilde psi_{i}$(each domian is $mathbf{R}^{n}$).



All functions $$varphi_{i}=frac{psi_{i}}{sum_{k=1}^{n}psi_{k}};(i=1,cdots,n)$$ are only constructed on $U$. $$ forall : xin U,: sum_{k=1}^{n}widetildepsi_{k}ne 0 ;;widetildepsi_{k}in C^{infty}(mathbf{R}^{n},mathbf{R}).$$ $$Longrightarrow varphi_{i}=frac{psi_{i}}{sum_{k=1}^{n}psi_{k}}=frac{widetildepsi_{i}}{sum_{k=1}^{n}widetildepsi_{k}}in C^{infty}(U,mathbf{R}) quad (i=1,2,cdots,n).$$



$textbf{2.}$



$fin C^{infty}(U,mathbf{R}),$ which value is $1$ on $A$ and $0$ outside of some closed set contained in $U$.(problem 2-26$^*$(d))



$\$Obviously,we have $fcdot varphi_{i}in C^{infty}(U,mathbf{R});(i=1,2,cdots,n).$ Note that each $fcdot varphi_{i}in C^{infty}(U,mathbf{R})$ is only constructed on $U:!$



$textbf{3.}$



Finally,we can extend $fcdot varphi_{i}$ from $U$ to the whole of $mathbf {R}^{n}$ smoothly.



In fact,since $supp(fcdotvarphi_{i})subset U$,
define $$widetilde{fcdot varphi_{i}}:=left{begin{matrix}
fcdot varphi_{i}& xin supp(fcdotvarphi_{i})\
0 & xnotin supp(fcdotvarphi_{i})
end{matrix}right.quad (i=1,2,cdots,n),$$
then we have $widetilde {fcdot varphi_{i}}in C^{infty}(mathbf{R}^{n},mathbf{R})$,$;widetilde {fcdot varphi_{i}}bigg|_{U}=fcdotvarphi_{i}$ and $fcdot varphi_{i}$ (each domain is $U$) can be smoothly extended to $widetilde {fcdot varphi_{i}}$ (each domian is $mathbf{R}^{n}).qquadqquadqquadqquadqquadqquadqquadqquadblacksquare$





A schematic diagram
(A schematic diagram concretising above elaboration)






share|cite|improve this answer











$endgroup$



Merry Christmas ! I am sorry to bother you.I revised my answer.Looking foward to your comments.



First of all, as Mr.John mentioned in his answer,the functions $varphi_{i}$ have already satisfied all of the conditions of Th 3-11 but except for $textit{(4)}$. A crucial question in Spivak's proof is: why the author "redundantly" required $fcdotvarphi_{i}$ rather than $varphi_{i}$ ?
In my way of thinking , in accord with problem 2-26$^*$(d),the author wanted to extend $varphi_{i}$ from its domain$- $an open subset in $mathbf {R}^{n}$$- $to the whole of $mathbf {R}^{n}$ smoothly and then guarantee such those extended functions $fcdot varphi_{i}$ satisfying all of four conditions.



The following is details:



Def. $;phi:X rightarrow mathbf {R}$ is a continuous real-valued function whose domain is an arbitrary set $X$ in $mathbf {R}^{n}$, the support of $phi$ is defined as the closure of the subset of $X$ where $phi$ is non-zero $i.e.,:$$supp(phi):=$the closure of the set $left{mathbf{x} in X: phi(mathbf{x})ne0 right }.$



Def. $;phiin C^{infty}(A,B)$ denotes $phi:(mathbf {R}^{n}supset )Arightarrow B(subsetmathbf {R}^{m})$ is a $C^{infty}$ function.



From the case 1,the compact sets $D_{i}(i=1,cdots,n)$ whose interiors cover $A$.In order to explain why the author modified $varphi_{i}$ with $fcdot varphi_{i}$ briefly,we can choose a specific open subset$- U:=bigcup_{i=1}^{n}int D_{i}$$- $to set forth.If in the simplest case we need $varphi_{i}$ multiplied by $f$,not to mention in most cases.



$textbf{1.}$



$psi_iin C^{infty}(U_i,mathbf{R}),$ which is positive on $D_i$ and $0$ outside of some closed set contained in $U_i$ .(problem 2-26$^*$(d))



Define $$widetilde psi_{i}:=left{begin{matrix}
psi_{i}& xin U_i\
0& xin {U_i}^{c}
end{matrix}right.quad (i=1,2,cdots,n),$$
then we have $widetilde psi_{i}in C^{infty}(mathbf{R}^{n},mathbf{R})$,$;widetilde psi_{i}bigg|_{U_{i}}=widetilde psi_{i}$ and $psi_{i}$ (each domain is $U_{i}$) can be smoothly extended to $widetilde psi_{i}$(each domian is $mathbf{R}^{n}$).



All functions $$varphi_{i}=frac{psi_{i}}{sum_{k=1}^{n}psi_{k}};(i=1,cdots,n)$$ are only constructed on $U$. $$ forall : xin U,: sum_{k=1}^{n}widetildepsi_{k}ne 0 ;;widetildepsi_{k}in C^{infty}(mathbf{R}^{n},mathbf{R}).$$ $$Longrightarrow varphi_{i}=frac{psi_{i}}{sum_{k=1}^{n}psi_{k}}=frac{widetildepsi_{i}}{sum_{k=1}^{n}widetildepsi_{k}}in C^{infty}(U,mathbf{R}) quad (i=1,2,cdots,n).$$



$textbf{2.}$



$fin C^{infty}(U,mathbf{R}),$ which value is $1$ on $A$ and $0$ outside of some closed set contained in $U$.(problem 2-26$^*$(d))



$\$Obviously,we have $fcdot varphi_{i}in C^{infty}(U,mathbf{R});(i=1,2,cdots,n).$ Note that each $fcdot varphi_{i}in C^{infty}(U,mathbf{R})$ is only constructed on $U:!$



$textbf{3.}$



Finally,we can extend $fcdot varphi_{i}$ from $U$ to the whole of $mathbf {R}^{n}$ smoothly.



In fact,since $supp(fcdotvarphi_{i})subset U$,
define $$widetilde{fcdot varphi_{i}}:=left{begin{matrix}
fcdot varphi_{i}& xin supp(fcdotvarphi_{i})\
0 & xnotin supp(fcdotvarphi_{i})
end{matrix}right.quad (i=1,2,cdots,n),$$
then we have $widetilde {fcdot varphi_{i}}in C^{infty}(mathbf{R}^{n},mathbf{R})$,$;widetilde {fcdot varphi_{i}}bigg|_{U}=fcdotvarphi_{i}$ and $fcdot varphi_{i}$ (each domain is $U$) can be smoothly extended to $widetilde {fcdot varphi_{i}}$ (each domian is $mathbf{R}^{n}).qquadqquadqquadqquadqquadqquadqquadqquadblacksquare$





A schematic diagram
(A schematic diagram concretising above elaboration)







share|cite|improve this answer














share|cite|improve this answer



share|cite|improve this answer








edited Dec 27 '18 at 7:03

























answered Dec 25 '18 at 5:28









user553010user553010

688




688












  • $begingroup$
    Thanks for elaborating, this makes sense. A few comments: 1. Actually the functions $varphi_i$ do not satisfy property 4 of the theorem which is why we need $f$ to kill it off near some parts of the boundary of $U_i$. 2. In your method, $U$ is defined slightly differently but the result is the same, support of $widetilde{fcdot varphi_{i}}$ is contained inside $U_i$ as required by property 4. I want to point out that this can't be made stronger to support contained in $D_i$. So I think I prefer the $U$ as defined by Spivak because it is easier to see the containment in $U_i$.
    $endgroup$
    – Pratyush Sarkar
    Dec 26 '18 at 23:06


















  • $begingroup$
    Thanks for elaborating, this makes sense. A few comments: 1. Actually the functions $varphi_i$ do not satisfy property 4 of the theorem which is why we need $f$ to kill it off near some parts of the boundary of $U_i$. 2. In your method, $U$ is defined slightly differently but the result is the same, support of $widetilde{fcdot varphi_{i}}$ is contained inside $U_i$ as required by property 4. I want to point out that this can't be made stronger to support contained in $D_i$. So I think I prefer the $U$ as defined by Spivak because it is easier to see the containment in $U_i$.
    $endgroup$
    – Pratyush Sarkar
    Dec 26 '18 at 23:06
















$begingroup$
Thanks for elaborating, this makes sense. A few comments: 1. Actually the functions $varphi_i$ do not satisfy property 4 of the theorem which is why we need $f$ to kill it off near some parts of the boundary of $U_i$. 2. In your method, $U$ is defined slightly differently but the result is the same, support of $widetilde{fcdot varphi_{i}}$ is contained inside $U_i$ as required by property 4. I want to point out that this can't be made stronger to support contained in $D_i$. So I think I prefer the $U$ as defined by Spivak because it is easier to see the containment in $U_i$.
$endgroup$
– Pratyush Sarkar
Dec 26 '18 at 23:06




$begingroup$
Thanks for elaborating, this makes sense. A few comments: 1. Actually the functions $varphi_i$ do not satisfy property 4 of the theorem which is why we need $f$ to kill it off near some parts of the boundary of $U_i$. 2. In your method, $U$ is defined slightly differently but the result is the same, support of $widetilde{fcdot varphi_{i}}$ is contained inside $U_i$ as required by property 4. I want to point out that this can't be made stronger to support contained in $D_i$. So I think I prefer the $U$ as defined by Spivak because it is easier to see the containment in $U_i$.
$endgroup$
– Pratyush Sarkar
Dec 26 '18 at 23:06


















draft saved

draft discarded




















































Thanks for contributing an answer to Mathematics Stack Exchange!


  • Please be sure to answer the question. Provide details and share your research!

But avoid



  • Asking for help, clarification, or responding to other answers.

  • Making statements based on opinion; back them up with references or personal experience.


Use MathJax to format equations. MathJax reference.


To learn more, see our tips on writing great answers.




draft saved


draft discarded














StackExchange.ready(
function () {
StackExchange.openid.initPostLogin('.new-post-login', 'https%3a%2f%2fmath.stackexchange.com%2fquestions%2f475060%2fpartition-of-unity-in-spivaks-calculus-on-manifolds%23new-answer', 'question_page');
}
);

Post as a guest















Required, but never shown





















































Required, but never shown














Required, but never shown












Required, but never shown







Required, but never shown

































Required, but never shown














Required, but never shown












Required, but never shown







Required, but never shown







Popular posts from this blog

Quarter-circle Tiles

build a pushdown automaton that recognizes the reverse language of a given pushdown automaton?

Mont Emei